Showing that monotone functions have at most countable discontinuities.












3












$begingroup$


I want to show that a map $F: mathbf{R} to mathbf{R}$ has at most countable discountinuities, if $F(x) leq F(y)$ whenever $x leq y$.



Here's the idea. Let's use standard notation $F(x^+), F(x^-)$ for upper and lower limits of $F$ around $x$. And let $D(F) = {x : F(x^+) neq F(x^-)}$, which is the set of discountinuities for a monotone function.
Now suppose that $D(F)$ is uncountable. Suppose $x, y in D(F)$ are distinct. Then $(F(x^-), F(x^+)) cap (F(y^-), F(y^+)) = emptyset$. This is because with $r = d(x, y)$, and assuming $x < y$, if $F(x^+) > F(y^-)$, this means that $inf_{x leq s < x + r/2} F(x) > sup_{y - r/2 < t leq y} F(t)$, which happens if and only if for some particular $x leq s < x + r/2$ and $y - r/2 < t leq y$, we have $F(s)> F(t)$. But this is a contradiction since as stated, $s leq t$. Hence we have $F(x^+) leq F(y^-)$, which means the intervals are disjoint as required.



We have shown that $D(F)$ uncountable implies the existence of uncountably many disjoint intervals $(F(x^-), F(x^+))$, where $x$ ranges over $D(F)$. But this can't happen because each open interval contains a distinct rational, which is a countable set.










share|cite|improve this question









$endgroup$












  • $begingroup$
    Your proof looks good. :)
    $endgroup$
    – Tki Deneb
    Dec 8 '18 at 20:59










  • $begingroup$
    Yes this is exactly the proof I learned. It might bear mentioning why $F(x^+)$ and $F(x^-)$ exist (which is easy) but this is right.
    $endgroup$
    – user25959
    Dec 8 '18 at 20:59
















3












$begingroup$


I want to show that a map $F: mathbf{R} to mathbf{R}$ has at most countable discountinuities, if $F(x) leq F(y)$ whenever $x leq y$.



Here's the idea. Let's use standard notation $F(x^+), F(x^-)$ for upper and lower limits of $F$ around $x$. And let $D(F) = {x : F(x^+) neq F(x^-)}$, which is the set of discountinuities for a monotone function.
Now suppose that $D(F)$ is uncountable. Suppose $x, y in D(F)$ are distinct. Then $(F(x^-), F(x^+)) cap (F(y^-), F(y^+)) = emptyset$. This is because with $r = d(x, y)$, and assuming $x < y$, if $F(x^+) > F(y^-)$, this means that $inf_{x leq s < x + r/2} F(x) > sup_{y - r/2 < t leq y} F(t)$, which happens if and only if for some particular $x leq s < x + r/2$ and $y - r/2 < t leq y$, we have $F(s)> F(t)$. But this is a contradiction since as stated, $s leq t$. Hence we have $F(x^+) leq F(y^-)$, which means the intervals are disjoint as required.



We have shown that $D(F)$ uncountable implies the existence of uncountably many disjoint intervals $(F(x^-), F(x^+))$, where $x$ ranges over $D(F)$. But this can't happen because each open interval contains a distinct rational, which is a countable set.










share|cite|improve this question









$endgroup$












  • $begingroup$
    Your proof looks good. :)
    $endgroup$
    – Tki Deneb
    Dec 8 '18 at 20:59










  • $begingroup$
    Yes this is exactly the proof I learned. It might bear mentioning why $F(x^+)$ and $F(x^-)$ exist (which is easy) but this is right.
    $endgroup$
    – user25959
    Dec 8 '18 at 20:59














3












3








3


1



$begingroup$


I want to show that a map $F: mathbf{R} to mathbf{R}$ has at most countable discountinuities, if $F(x) leq F(y)$ whenever $x leq y$.



Here's the idea. Let's use standard notation $F(x^+), F(x^-)$ for upper and lower limits of $F$ around $x$. And let $D(F) = {x : F(x^+) neq F(x^-)}$, which is the set of discountinuities for a monotone function.
Now suppose that $D(F)$ is uncountable. Suppose $x, y in D(F)$ are distinct. Then $(F(x^-), F(x^+)) cap (F(y^-), F(y^+)) = emptyset$. This is because with $r = d(x, y)$, and assuming $x < y$, if $F(x^+) > F(y^-)$, this means that $inf_{x leq s < x + r/2} F(x) > sup_{y - r/2 < t leq y} F(t)$, which happens if and only if for some particular $x leq s < x + r/2$ and $y - r/2 < t leq y$, we have $F(s)> F(t)$. But this is a contradiction since as stated, $s leq t$. Hence we have $F(x^+) leq F(y^-)$, which means the intervals are disjoint as required.



We have shown that $D(F)$ uncountable implies the existence of uncountably many disjoint intervals $(F(x^-), F(x^+))$, where $x$ ranges over $D(F)$. But this can't happen because each open interval contains a distinct rational, which is a countable set.










share|cite|improve this question









$endgroup$




I want to show that a map $F: mathbf{R} to mathbf{R}$ has at most countable discountinuities, if $F(x) leq F(y)$ whenever $x leq y$.



Here's the idea. Let's use standard notation $F(x^+), F(x^-)$ for upper and lower limits of $F$ around $x$. And let $D(F) = {x : F(x^+) neq F(x^-)}$, which is the set of discountinuities for a monotone function.
Now suppose that $D(F)$ is uncountable. Suppose $x, y in D(F)$ are distinct. Then $(F(x^-), F(x^+)) cap (F(y^-), F(y^+)) = emptyset$. This is because with $r = d(x, y)$, and assuming $x < y$, if $F(x^+) > F(y^-)$, this means that $inf_{x leq s < x + r/2} F(x) > sup_{y - r/2 < t leq y} F(t)$, which happens if and only if for some particular $x leq s < x + r/2$ and $y - r/2 < t leq y$, we have $F(s)> F(t)$. But this is a contradiction since as stated, $s leq t$. Hence we have $F(x^+) leq F(y^-)$, which means the intervals are disjoint as required.



We have shown that $D(F)$ uncountable implies the existence of uncountably many disjoint intervals $(F(x^-), F(x^+))$, where $x$ ranges over $D(F)$. But this can't happen because each open interval contains a distinct rational, which is a countable set.







real-analysis proof-verification






share|cite|improve this question













share|cite|improve this question











share|cite|improve this question




share|cite|improve this question










asked Dec 8 '18 at 20:45









Drew BradyDrew Brady

721315




721315












  • $begingroup$
    Your proof looks good. :)
    $endgroup$
    – Tki Deneb
    Dec 8 '18 at 20:59










  • $begingroup$
    Yes this is exactly the proof I learned. It might bear mentioning why $F(x^+)$ and $F(x^-)$ exist (which is easy) but this is right.
    $endgroup$
    – user25959
    Dec 8 '18 at 20:59


















  • $begingroup$
    Your proof looks good. :)
    $endgroup$
    – Tki Deneb
    Dec 8 '18 at 20:59










  • $begingroup$
    Yes this is exactly the proof I learned. It might bear mentioning why $F(x^+)$ and $F(x^-)$ exist (which is easy) but this is right.
    $endgroup$
    – user25959
    Dec 8 '18 at 20:59
















$begingroup$
Your proof looks good. :)
$endgroup$
– Tki Deneb
Dec 8 '18 at 20:59




$begingroup$
Your proof looks good. :)
$endgroup$
– Tki Deneb
Dec 8 '18 at 20:59












$begingroup$
Yes this is exactly the proof I learned. It might bear mentioning why $F(x^+)$ and $F(x^-)$ exist (which is easy) but this is right.
$endgroup$
– user25959
Dec 8 '18 at 20:59




$begingroup$
Yes this is exactly the proof I learned. It might bear mentioning why $F(x^+)$ and $F(x^-)$ exist (which is easy) but this is right.
$endgroup$
– user25959
Dec 8 '18 at 20:59










1 Answer
1






active

oldest

votes


















0












$begingroup$

I assume your question is, is this proof valid? The answer is yes!



To be clearer, you should probably point out that $x + r/2 = y - r/2$, which is how you get $s < x + r/2 = y - r/2 < t$ and therefore $s le t$ (you say this is true "as stated", but I don't see you stating it).



You might also want to say more about why a discontinuity in a monotone function must have $F(x^+) ne F(x^-)$.






share|cite|improve this answer









$endgroup$













    Your Answer





    StackExchange.ifUsing("editor", function () {
    return StackExchange.using("mathjaxEditing", function () {
    StackExchange.MarkdownEditor.creationCallbacks.add(function (editor, postfix) {
    StackExchange.mathjaxEditing.prepareWmdForMathJax(editor, postfix, [["$", "$"], ["\\(","\\)"]]);
    });
    });
    }, "mathjax-editing");

    StackExchange.ready(function() {
    var channelOptions = {
    tags: "".split(" "),
    id: "69"
    };
    initTagRenderer("".split(" "), "".split(" "), channelOptions);

    StackExchange.using("externalEditor", function() {
    // Have to fire editor after snippets, if snippets enabled
    if (StackExchange.settings.snippets.snippetsEnabled) {
    StackExchange.using("snippets", function() {
    createEditor();
    });
    }
    else {
    createEditor();
    }
    });

    function createEditor() {
    StackExchange.prepareEditor({
    heartbeatType: 'answer',
    autoActivateHeartbeat: false,
    convertImagesToLinks: true,
    noModals: true,
    showLowRepImageUploadWarning: true,
    reputationToPostImages: 10,
    bindNavPrevention: true,
    postfix: "",
    imageUploader: {
    brandingHtml: "Powered by u003ca class="icon-imgur-white" href="https://imgur.com/"u003eu003c/au003e",
    contentPolicyHtml: "User contributions licensed under u003ca href="https://creativecommons.org/licenses/by-sa/3.0/"u003ecc by-sa 3.0 with attribution requiredu003c/au003e u003ca href="https://stackoverflow.com/legal/content-policy"u003e(content policy)u003c/au003e",
    allowUrls: true
    },
    noCode: true, onDemand: true,
    discardSelector: ".discard-answer"
    ,immediatelyShowMarkdownHelp:true
    });


    }
    });














    draft saved

    draft discarded


















    StackExchange.ready(
    function () {
    StackExchange.openid.initPostLogin('.new-post-login', 'https%3a%2f%2fmath.stackexchange.com%2fquestions%2f3031620%2fshowing-that-monotone-functions-have-at-most-countable-discontinuities%23new-answer', 'question_page');
    }
    );

    Post as a guest















    Required, but never shown

























    1 Answer
    1






    active

    oldest

    votes








    1 Answer
    1






    active

    oldest

    votes









    active

    oldest

    votes






    active

    oldest

    votes









    0












    $begingroup$

    I assume your question is, is this proof valid? The answer is yes!



    To be clearer, you should probably point out that $x + r/2 = y - r/2$, which is how you get $s < x + r/2 = y - r/2 < t$ and therefore $s le t$ (you say this is true "as stated", but I don't see you stating it).



    You might also want to say more about why a discontinuity in a monotone function must have $F(x^+) ne F(x^-)$.






    share|cite|improve this answer









    $endgroup$


















      0












      $begingroup$

      I assume your question is, is this proof valid? The answer is yes!



      To be clearer, you should probably point out that $x + r/2 = y - r/2$, which is how you get $s < x + r/2 = y - r/2 < t$ and therefore $s le t$ (you say this is true "as stated", but I don't see you stating it).



      You might also want to say more about why a discontinuity in a monotone function must have $F(x^+) ne F(x^-)$.






      share|cite|improve this answer









      $endgroup$
















        0












        0








        0





        $begingroup$

        I assume your question is, is this proof valid? The answer is yes!



        To be clearer, you should probably point out that $x + r/2 = y - r/2$, which is how you get $s < x + r/2 = y - r/2 < t$ and therefore $s le t$ (you say this is true "as stated", but I don't see you stating it).



        You might also want to say more about why a discontinuity in a monotone function must have $F(x^+) ne F(x^-)$.






        share|cite|improve this answer









        $endgroup$



        I assume your question is, is this proof valid? The answer is yes!



        To be clearer, you should probably point out that $x + r/2 = y - r/2$, which is how you get $s < x + r/2 = y - r/2 < t$ and therefore $s le t$ (you say this is true "as stated", but I don't see you stating it).



        You might also want to say more about why a discontinuity in a monotone function must have $F(x^+) ne F(x^-)$.







        share|cite|improve this answer












        share|cite|improve this answer



        share|cite|improve this answer










        answered Dec 8 '18 at 21:02









        Hew WolffHew Wolff

        2,260716




        2,260716






























            draft saved

            draft discarded




















































            Thanks for contributing an answer to Mathematics Stack Exchange!


            • Please be sure to answer the question. Provide details and share your research!

            But avoid



            • Asking for help, clarification, or responding to other answers.

            • Making statements based on opinion; back them up with references or personal experience.


            Use MathJax to format equations. MathJax reference.


            To learn more, see our tips on writing great answers.




            draft saved


            draft discarded














            StackExchange.ready(
            function () {
            StackExchange.openid.initPostLogin('.new-post-login', 'https%3a%2f%2fmath.stackexchange.com%2fquestions%2f3031620%2fshowing-that-monotone-functions-have-at-most-countable-discontinuities%23new-answer', 'question_page');
            }
            );

            Post as a guest















            Required, but never shown





















































            Required, but never shown














            Required, but never shown












            Required, but never shown







            Required, but never shown

































            Required, but never shown














            Required, but never shown












            Required, but never shown







            Required, but never shown







            Popular posts from this blog

            Biblatex bibliography style without URLs when DOI exists (in Overleaf with Zotero bibliography)

            ComboBox Display Member on multiple fields

            Is it possible to collect Nectar points via Trainline?